Proving $T:X\to X$ is not a Contraction on $X=[1, \infty)$

  • MHB
  • Thread starter ozkan12
  • Start date
  • Tags
    Contraction
In summary: Basically, what this shows is that your map might be a contraction, but it's not a certain thing. You might have to go to the limit to see if it actually is.In summary, the function $T$ is not a contraction because it does not have a unique fixed point.
  • #1
ozkan12
149
0
Let $X=[1,\infty)$ and $T:X\to X$. Define $T=x+\frac{1}{x}$...Please show that T is not a contraction
 
Physics news on Phys.org
  • #2
I'm a little confused.
$$T'(x)=1-\frac{1}{x^2} <1,$$
showing that $T$ is a contraction. Are you using a different metric? Something exotic?
 
  • #3
Dear,

How this shows that T is a contraction ? I don't understand...We use usual metric..

Also, I want to show that T is not a contraction
 
  • #4
If the derivative is strictly smaller than 1 everywhere in the space, this suggests that the mapping is a contraction. The criteria for a contraction mapping is that $|T(x)-T(y)| \le k |x-y|$, where $0\le k <1$. Now, if we assume $x\not=y$, this is like saying that
$$\frac{|T(x)-T(y)|}{|x-y|} \le k.$$
The LHS, you can see, looks a lot like a derivative. By the Mean Value Theorem, if the derivative is always strictly less than 1, then the LHS (slope of a secant line) must always be less than 1. Now I will say this: the value of $k$, it seems to me, is not constant on the entire space. I'm not sure you could have a particular $k$ that works everywhere. But my intuition tells me that you're never going to be able to produce a counterexample for the basic inequality of $|T(x)-T(y)| \le k |x-y|$. That's what you'd have to do to show that $T$ is not a contraction.

[EDIT] See below for a correction.
 
  • #6
Yes, I see now where you're going with this. I was too hasty in my previous post. The mapping $T$ isn't a contraction, even though it always maps two elements closer together than they were before. And that's because there isn't any one $0\le k<1$ such that the slopes of secant lines are always less than or equal to $k$. But you have to go to the limit to get a counterexample, which makes sense. $T$ is about as close to a contraction as you can get without actually being one.
 
  • #7
Dear,

I didnt understand...How can you explain this by limit ?
 
  • #8
Here's a link to information on the contraction mapping principle -- https://en.wikipedia.org/wiki/Banach_fixed-point_theorem.
By this theorem, if your map were a contraction, there would be a unique fixed point. But of course the equation $x=x+1/x$ clearly has no solutions. If you read the information in the link, your function is actually briefly discussed.
 

Related to Proving $T:X\to X$ is not a Contraction on $X=[1, \infty)$

1. How do you define a contraction on a set?

A contraction on a set is a function that maps elements from the set to itself and satisfies the condition that the distance between the images of any two points is always less than the distance between the original points. In other words, a contraction shrinks the distance between points in the set.

2. Why is proving that a function is not a contraction important?

Proving that a function is not a contraction is important because it allows us to determine whether or not the function has a fixed point. If a function is a contraction, it is guaranteed to have a unique fixed point. However, if a function is not a contraction, it may have multiple fixed points or no fixed points at all.

3. How do you prove that a function is not a contraction on a given set?

To prove that a function is not a contraction on a given set, we need to show that there exists at least one pair of points in the set whose images have a distance greater than or equal to the distance between the original points. This can be done by finding specific points in the set and calculating their images under the function.

4. Can a function be a contraction on one set but not on another?

Yes, a function can be a contraction on one set but not on another. The definition of a contraction depends on the set that the function is being mapped to. Therefore, a function may satisfy the contraction condition on one set but not on another.

5. How is proving a function is not a contraction related to the Banach Fixed Point Theorem?

The Banach Fixed Point Theorem states that if a function is a contraction on a complete metric space, then it has a unique fixed point in that space. Therefore, by proving that a function is not a contraction on a specific set, we are also showing that the set is not a complete metric space, and the Banach Fixed Point Theorem does not apply. This helps us in understanding the properties of the set and the behavior of the function on that set.

Similar threads

  • Topology and Analysis
Replies
4
Views
406
  • Topology and Analysis
Replies
3
Views
1K
Replies
4
Views
427
Replies
2
Views
1K
  • Topology and Analysis
Replies
1
Views
1K
  • Topology and Analysis
Replies
7
Views
2K
Replies
1
Views
1K
  • Topology and Analysis
Replies
2
Views
1K
  • Topology and Analysis
Replies
11
Views
1K
  • Topology and Analysis
Replies
4
Views
2K
Back
Top